site stats

H t -16t2 + 8t + k. what is the value of k

Web28 mei 2024 · Answer: The value of k is 2. Explanation: From the figure it is noticed that the triangle LMN, LON and MNO are right angle triangle. According to the pythagoras theorem, Use pythagoras theorem in triangle LON. Use pythagoras theorem in triangle MON. ..... (1) Use pythagoras theorem in triangle LMN. .... (2) From equation (1) and (2), we get Web20 dec. 2024 · Solution. The acceleration vector of the enemy missile is. ae(t) = − 9.8ˆj. Integrating, we get the velocity vector. ve(t) = v1ˆi + (v2 − 9.8t)ˆj. Setting t = 0 and using the initial velocity of the enemy missile gives. ve(t) = − 30ˆi + (3 − 9.8t)ˆj. Now integrate again to find the position function.

K-Value, U-Value, R-Value, C-Value - Insulation Outlook Magazine

Web11 jan. 2024 · Since the other posters have already correctly answered the question, I won't rehash any of that work here. Instead, I'll elaborate a bit more on the Exponent rules involved. As a category, you won't see too many Exponent rule prompts on Test Day (probably 2-3, not counting squared-terms or Quadratics). WebFind the Maximum/Minimum Value h(t)=-16t^2+8t. Step 1. The maximum of a quadratic function occurs at . If is negative, the maximum value of the function is . occurs at . Step … bird\\u0027s wife https://2brothers2chefs.com

If K is an integer and 2 < K < 8, what is the value of K

Web8 dec. 2024 · If k is an integer and 2 < k < 8, what is the value of k ? (1) k is a factor of 30. (2) k is a factor of 12 The answer given is E. from statement 1 : k can be 2,3,5. k>2, so 3 or 5. NS from statement 2 : k can be 2,3. k>2, so 3. Sufficient Hence my answer was B. Am i missing something You probably misunderstood and worked for only primes. WebThe equation: I d = K 2 ( V g s − V t) 2 where K = μ C o x W L. describes the relation between I d and V g s when K and V t are known and when the MOSFET operates in … dance of the four swans piano

What is k in Statistics? - Speed Test Demon

Category:What is k in Statistics? - Speed Test Demon

Tags:H t -16t2 + 8t + k. what is the value of k

H t -16t2 + 8t + k. what is the value of k

If mn = K, what is the value of K ? : Data Sufficiency (DS)

Web29 jun. 2024 · Find an answer to your question What is the value of k? k = 28 k = 29 k = 31 k = 42. katanalopez katanalopez 06/29/2024 Mathematics High School answered • expert verified What is the value of k? k = 28 k = 29 k = 31 k = 42 See answers Advertisement Advertisement calculista calculista we know that-----&gt; by supplementary angles. Webh(t) = - 16t² + 8t + k. ANSWER TO QUESTION : 1. By the given condition h(0) = 48. Again h(0) = - 16 × 0² + 8 × 0 + k. ⇒ 48 = k. ⇒ k = 48. Hence the required value of k = 48. …

H t -16t2 + 8t + k. what is the value of k

Did you know?

Web2 dec. 2024 · $\begingroup$ In the days before calculators, people published tables of logarithms (and "antilogarithms"), and you would look for $.79$ in one of those tables, and get the answer that way. Or you would take out your slide rule and use the log scale on it. Calculators have made log tables and slide rules obsolete. $\endgroup$ – Gerry Myerson Web2 feb. 2024 · This temperature is represented by the x -intercept of the line, that is, the value of T for which Δ G is zero: ΔG = 0 = ΔH − TΔS. T = ΔH ΔS. And so, saying a process is spontaneous at “high” or “low” temperatures means the temperature is above or below, respectively, that temperature at which Δ G for the process is zero.

Web17 jun. 2024 · The value of k is: k = 29. Step-by-step explanation: We know that when two angles lie on a straight line then the sum of the two angles is equal to 180 degree. Here we have two angles on a straight line as: first angle = (5k-3)° and second angle= (9+k)° Hence, we have: (5k-3)°+(9+k)°=180° i.e. 5k-3+9+k=180 Web7 jun. 2024 · The zero product property states that if two terms being multiplied have a product of zero, then at least one of them must be zero. In our case, this means we can set both (5k - 3) and (9 + k) equal to 0: 5k - 3 = 0 5k = 3 k = 3/5 9 + k = 0 k = -9 The value of k is 3/5 and -9. Advertisement New questions in Mathematics Please answer the question

WebThinking about velocity, speed, and definite integrals. Say a particle moves in a straight line with velocity v (t)=5-t v(t) = 5−t meters per second, where t t is time in seconds. When the velocity is positive it means the particle is moving forward along the line, and when the velocity is negative it means the particle is moving backwards. Web9 mei 2012 · mn = K, find value of K Stmt 1: m+n = 2mn (3 variables 2 equations - multiple solutions possible) Stmt 2: on solving you get mn=0 =&gt; K=0 (thus sufficient) bumpbot Non-Human User. Joined: 09 Sep 2013 . Posts: 27323. Own Kudos : …

Web30 jul. 2016 · Step-by-step explanation: Given : h (t) , g (t) ,k (t), h (t). To find : Which function is a quadratic function . Solution : We have given that different function , …

Web18 feb. 2024 · Viewed 55 times. 0. Whilst using this figure: enter image description here. It's a sketch showing the part of the curve C with equation: y = ( 2 x − 1) 3 4, x ≥ 1 2. The … bird\u0027s underwater crystal riverWeb8 dec. 2024 · Since k is an integer between 2 and 8,the possible values of k are 3,5 and 6. Since we have no definite value of k, the statement stands insufficient. (insufficient) … dance of the four windsWebThe common denominator of the two fractions is 1 Adding (-16/1)+ (16/1) gives 0/1 So adding to both sides we finally get : t2-8t+16 = 0 Adding 16 has completed the left hand side into a perfect square : t2-8t+16 = (t-4) • (t-4) = (t-4)2 Things which are equal to the same thing are also equal to one another. Since t2-8t+16 = 0 and t2-8t+16 = (t-4)2 bird\\u0027s surf shed